Art of Problem Solving
Art of Problem Solving
SIGN IN REGISTER
Problem
Real numbers , , and are chosen independently
and at random from the interval for some
positive integer . The probability that no two of , ,
Solution 1
Since are all reals located in , the
number of choices for each one is continuous so we
use geometric probability.
WLOG(Without loss of generality), assume that
. Then the set of points
is a tetrahedron, or a triangular pyramid. The point
distributes uniformly in this region. If this is
not easy to understand, read Solution II.
The altitude of the tetrahedron is and the base is an
isosceles right triangle with a leg length . The
So the probability is .
. So .
So the answer is .
Solution 2
Because , , and are chosen independently and at
random from the interval , which means that ,
, and distributes uniformly and independently in
the interval . So the point distributes
uniformly in the cubic , as shown in
the figure below. The volume of this cubic is .
.
The region can be generated by cutting off 3 slices
corresponding to , , and
, respectively, from the cubic.
After cutting off a slice corresponding to ,
we get two triangular prisms, as shown in the figure.
is = .
The answer is .
Let , i.e., , so
, or , hence
calculator.
Since = , so =
= .
Estimation of .
Since , so .
The mean of 1 and 1.5 with one decimal digit is about
1.3 .
As , so
.
The mean of 1 and 1.3 with one decimal digit is about
1.2.
As , so
.
Estimation of .
As , so , then
.
As , so
.
The mean of 1.5 and 1.69 with one decimal digit is
about 1.6.
As
, so .
Then
, i.e., ,
Appendix
This solution is motivated by the suggestive formula
Solution 3
If , , and are separated by at least one, then by
subtracting the minimum space between the three
variables (which is ), , , and can be chosen
randomly in the interval . The probability is
Solution 4
Imagine Points , , as the "starting points" of three
"blocks" of real numbers that have length . We are
just trying to find the probability that those three
"blocks" do not overlap. To do this we can set each
unit of into equal little increments, and take the
limit of the probability as approaches . (This is
because there are indefinitely many real numbers in a
given interval.) We see that the total number of
arrangements for the three blocks of , without
considering the rule that no two blocks shall overlap is
. We see that the number of ways to
arrange the three blocks such that no three of them
least value of as .
-fidgetboss_4000
See Also
2012 AMC 10A (Problems • Answer Key •
Resources)
Preceded by Followed by
Problem 24 Last Problem
1 • 2 • 3 • 4 • 5 • 6 • 7 • 8 • 9 • 10 • 11 • 12 •
13 • 14 • 15 • 16 • 17 • 18 • 19 • 20 • 21 • 22
• 23 • 24 • 25
All AMC 10 Problems and Solutions
Mathematics Competitions.
SITE INFO
Terms
Privacy
Contact Us
FOLLOW US